How Can You Compute Multivariable Limits in Mathematica?

Click For Summary
To compute multivariable limits in Mathematica, users can encounter challenges, particularly when limits are undefined due to path dependency, as seen in the example of lim_{(x,y)→(0,0)}(x^2/y). The limit does not exist because it varies based on the approach to (0,0), with different paths yielding different results. For valid limits, Mathematica can be used effectively, and contour plots are recommended to visually assess limit existence before calculation. Users are encouraged to explore specific functions to approach the limit correctly. Understanding these concepts is crucial for effectively using a Computer Algebra System (CAS) for multivariable calculus.
springo
Messages
125
Reaction score
0

Homework Statement


I would like to be able to compute multivariable limits with a CAS. I have Mathematica, Derive and an HP 50g.
For example, how would I compute:
\lim_{(x,y)\rightarrow(0,0)}\frac{x^2}{y}

Homework Equations



The Attempt at a Solution


On Mathematica I tried:
Code:
Limit[x^2/y, {x, y} -> {0, 0}]
(doesn't work)
Code:
Limit[x^2/y /. {y -> f[x]}, x -> 0]
(I think this could work, but I need to figure out what f[x] has to be)

Thanks for your help.
 
Physics news on Phys.org
The limit in your example does not exist (it is undefined), because it depends on how you approach (0,0). For example, f(t,t)->0 as t->0 from the right, but f(sqrt(t),t)->1 as t->0 from the right.
 
yyat said:
The limit in your example does not exist (it is undefined), because it depends on how you approach (0,0). For example, f(t,t)->0 as t->0 from the right, but f(sqrt(t),t)->1 as t->0 from the right.
OK, sorry for choosing a wrong example, I'm just starting multivariable calculus. Anyway for existing limits, is there any way to compute them using a CAS?
 
Here's a nice little example using mathematica. Contour Plots are typically a good way to infer whether a limit exists before you go ahead and calculate it.
 
Question: A clock's minute hand has length 4 and its hour hand has length 3. What is the distance between the tips at the moment when it is increasing most rapidly?(Putnam Exam Question) Answer: Making assumption that both the hands moves at constant angular velocities, the answer is ## \sqrt{7} .## But don't you think this assumption is somewhat doubtful and wrong?

Similar threads

  • · Replies 10 ·
Replies
10
Views
2K
Replies
5
Views
2K
  • · Replies 6 ·
Replies
6
Views
2K
  • · Replies 8 ·
Replies
8
Views
2K
  • · Replies 4 ·
Replies
4
Views
1K
  • · Replies 2 ·
Replies
2
Views
1K
  • · Replies 20 ·
Replies
20
Views
2K
  • · Replies 2 ·
Replies
2
Views
1K
Replies
24
Views
3K
  • · Replies 3 ·
Replies
3
Views
1K